57.3.22 To win democratic

Accounts PlayableAccounts Playable Live Sage
Absolutely clueless on this one. I've probably watched the video 5 times, and I just don't see how B weakens the argument that is in the passage. Also, can someone look at my reasoning for A?

Win democratic elections not fully subsidized--->poor candidates supported by rich. As a result, it could be true that the poor candidates will compromise their positions to win the rich guy's support. However, the proportion of rich people in all of the political parties is the same as their proportion in the population. Thus, the belief that it could be true that poor candidates will compromise their position to win the rich people's support is wrong.

What I am looking for: The premise doesn't support the conclusion at all since we don't know whether being proportionally equal nullifies the pressure to conform to the rich guy's opinion. For example, say that rich people are 1% of the population in all political parties; they must then be 1% of the total population. It might be reasonable to say that the rich don't have that big of an influence on policy. Now, what if the proportion was 99%? The rich might have a huge say in policy! Thus, the premise could go both ways in either providing support for the conclusion or not.

Answer A: This is what I chose, but I still am very unsure why it is wrong. I think the argument does fail to address this answer choice. Maybe it is wrong because of the word "primary?" I'm not so sure though since we are usually supposed to accept the answer choices as true. If it is the case that the "primary" function of a party still may not negate the influence of wealth, then doesn't this paraphrase the flaw very well? To me, this is hinting at the idea that the proportion of the wealthy might be so high (or so influential) in political parties that it doesn't matter which one the poor candidate choose, they will have to conform to the wealthy point of view's party.

Say that the word "primary" is the reason why this is wrong (which I am not really sure why it would be), what if this answer choice said "a function of political parties...?" Would it be correct then?

Answer B: Like I said earlier, I just don't see how this weakens the argument. I do think it weakens an argument, just not the one in the passage. Here is my breakdown of this answer choice: say the poor candidate believes, "Every person who takes the LSAT should get a 180 and full ride to Yale" but the Democrats and Republicans both think "absolutely not" (the positions of the parties is way less varied than the position of the candidate). Then sure, joining a political party would compromise the poor candidate's views. But, that isn't the argument in the passage. The argument is that the "possibility of a poor candidate compromising his views to win the support OF THE RICH [not the political party] is not true." How are these two the same argument? In other words, if this is true, isn't the LSAT equivocating between the views of the "political parties" and the views of the "wealthy patrons?"

Answer C: We don't care about government subsidized elections.

Answer D: We don't care about wealthy candidates.

Answer E: We don't care about other flaws.
Sign In or Register to comment.